Is there any well-ordered uncountable set of real numbers under the original ordering?











up vote
2
down vote

favorite
1












I know that the usual ordering of $mathbb R$ is not a well-ordering but is there an uncountable $Ssubset mathbb R$ such that S is well-ordered by $<_mathbb R$?



Intuitively I'd say there is no such set but intuitively I'd also say there is no well-ordered uncountable set at all, which is obviously wrong. I still struggle to grasp the idea of an uncountable, well-ordered set.










share|cite|improve this question


















  • 2




    FYI, a frequently asked question on the Indiana University (Bloomington) Ph.D. real variables qualifying exams back in the late 1960s to the mid 1970s was to prove that any well-ordered subset of the reals is countable. (Sometimes the question was whether an uncountable well-ordered subset of the reals exists, and you were to prove your answer.)
    – Dave L. Renfro
    Apr 22 '16 at 17:44








  • 2




    @Matt Dyer: akkarin is asking whether there exists such a set on the real line, not whether there exists such a set somewhere.
    – Dave L. Renfro
    Apr 22 '16 at 17:45






  • 1




    Hint: You can get a pairwise disjoint collection of open intervals by picking one open interval lying between each of the points and the next point of the well ordered set.
    – Dave L. Renfro
    Apr 22 '16 at 17:51








  • 1




    @Matt Dyer: True, but the ordering you get may not have the numbers being in the same order as they appear on the number line, which is what akkarin wants.
    – Dave L. Renfro
    Apr 22 '16 at 17:52






  • 2




    Why the vote to close? It's possible this is a duplicate (I vaguely remember the same question being asked earlier), but it's certainly not off-topic.
    – Noah Schweber
    Apr 22 '16 at 18:52















up vote
2
down vote

favorite
1












I know that the usual ordering of $mathbb R$ is not a well-ordering but is there an uncountable $Ssubset mathbb R$ such that S is well-ordered by $<_mathbb R$?



Intuitively I'd say there is no such set but intuitively I'd also say there is no well-ordered uncountable set at all, which is obviously wrong. I still struggle to grasp the idea of an uncountable, well-ordered set.










share|cite|improve this question


















  • 2




    FYI, a frequently asked question on the Indiana University (Bloomington) Ph.D. real variables qualifying exams back in the late 1960s to the mid 1970s was to prove that any well-ordered subset of the reals is countable. (Sometimes the question was whether an uncountable well-ordered subset of the reals exists, and you were to prove your answer.)
    – Dave L. Renfro
    Apr 22 '16 at 17:44








  • 2




    @Matt Dyer: akkarin is asking whether there exists such a set on the real line, not whether there exists such a set somewhere.
    – Dave L. Renfro
    Apr 22 '16 at 17:45






  • 1




    Hint: You can get a pairwise disjoint collection of open intervals by picking one open interval lying between each of the points and the next point of the well ordered set.
    – Dave L. Renfro
    Apr 22 '16 at 17:51








  • 1




    @Matt Dyer: True, but the ordering you get may not have the numbers being in the same order as they appear on the number line, which is what akkarin wants.
    – Dave L. Renfro
    Apr 22 '16 at 17:52






  • 2




    Why the vote to close? It's possible this is a duplicate (I vaguely remember the same question being asked earlier), but it's certainly not off-topic.
    – Noah Schweber
    Apr 22 '16 at 18:52













up vote
2
down vote

favorite
1









up vote
2
down vote

favorite
1






1





I know that the usual ordering of $mathbb R$ is not a well-ordering but is there an uncountable $Ssubset mathbb R$ such that S is well-ordered by $<_mathbb R$?



Intuitively I'd say there is no such set but intuitively I'd also say there is no well-ordered uncountable set at all, which is obviously wrong. I still struggle to grasp the idea of an uncountable, well-ordered set.










share|cite|improve this question













I know that the usual ordering of $mathbb R$ is not a well-ordering but is there an uncountable $Ssubset mathbb R$ such that S is well-ordered by $<_mathbb R$?



Intuitively I'd say there is no such set but intuitively I'd also say there is no well-ordered uncountable set at all, which is obviously wrong. I still struggle to grasp the idea of an uncountable, well-ordered set.







set-theory order-theory






share|cite|improve this question













share|cite|improve this question











share|cite|improve this question




share|cite|improve this question










asked Apr 22 '16 at 17:29









akkarin

668311




668311








  • 2




    FYI, a frequently asked question on the Indiana University (Bloomington) Ph.D. real variables qualifying exams back in the late 1960s to the mid 1970s was to prove that any well-ordered subset of the reals is countable. (Sometimes the question was whether an uncountable well-ordered subset of the reals exists, and you were to prove your answer.)
    – Dave L. Renfro
    Apr 22 '16 at 17:44








  • 2




    @Matt Dyer: akkarin is asking whether there exists such a set on the real line, not whether there exists such a set somewhere.
    – Dave L. Renfro
    Apr 22 '16 at 17:45






  • 1




    Hint: You can get a pairwise disjoint collection of open intervals by picking one open interval lying between each of the points and the next point of the well ordered set.
    – Dave L. Renfro
    Apr 22 '16 at 17:51








  • 1




    @Matt Dyer: True, but the ordering you get may not have the numbers being in the same order as they appear on the number line, which is what akkarin wants.
    – Dave L. Renfro
    Apr 22 '16 at 17:52






  • 2




    Why the vote to close? It's possible this is a duplicate (I vaguely remember the same question being asked earlier), but it's certainly not off-topic.
    – Noah Schweber
    Apr 22 '16 at 18:52














  • 2




    FYI, a frequently asked question on the Indiana University (Bloomington) Ph.D. real variables qualifying exams back in the late 1960s to the mid 1970s was to prove that any well-ordered subset of the reals is countable. (Sometimes the question was whether an uncountable well-ordered subset of the reals exists, and you were to prove your answer.)
    – Dave L. Renfro
    Apr 22 '16 at 17:44








  • 2




    @Matt Dyer: akkarin is asking whether there exists such a set on the real line, not whether there exists such a set somewhere.
    – Dave L. Renfro
    Apr 22 '16 at 17:45






  • 1




    Hint: You can get a pairwise disjoint collection of open intervals by picking one open interval lying between each of the points and the next point of the well ordered set.
    – Dave L. Renfro
    Apr 22 '16 at 17:51








  • 1




    @Matt Dyer: True, but the ordering you get may not have the numbers being in the same order as they appear on the number line, which is what akkarin wants.
    – Dave L. Renfro
    Apr 22 '16 at 17:52






  • 2




    Why the vote to close? It's possible this is a duplicate (I vaguely remember the same question being asked earlier), but it's certainly not off-topic.
    – Noah Schweber
    Apr 22 '16 at 18:52








2




2




FYI, a frequently asked question on the Indiana University (Bloomington) Ph.D. real variables qualifying exams back in the late 1960s to the mid 1970s was to prove that any well-ordered subset of the reals is countable. (Sometimes the question was whether an uncountable well-ordered subset of the reals exists, and you were to prove your answer.)
– Dave L. Renfro
Apr 22 '16 at 17:44






FYI, a frequently asked question on the Indiana University (Bloomington) Ph.D. real variables qualifying exams back in the late 1960s to the mid 1970s was to prove that any well-ordered subset of the reals is countable. (Sometimes the question was whether an uncountable well-ordered subset of the reals exists, and you were to prove your answer.)
– Dave L. Renfro
Apr 22 '16 at 17:44






2




2




@Matt Dyer: akkarin is asking whether there exists such a set on the real line, not whether there exists such a set somewhere.
– Dave L. Renfro
Apr 22 '16 at 17:45




@Matt Dyer: akkarin is asking whether there exists such a set on the real line, not whether there exists such a set somewhere.
– Dave L. Renfro
Apr 22 '16 at 17:45




1




1




Hint: You can get a pairwise disjoint collection of open intervals by picking one open interval lying between each of the points and the next point of the well ordered set.
– Dave L. Renfro
Apr 22 '16 at 17:51






Hint: You can get a pairwise disjoint collection of open intervals by picking one open interval lying between each of the points and the next point of the well ordered set.
– Dave L. Renfro
Apr 22 '16 at 17:51






1




1




@Matt Dyer: True, but the ordering you get may not have the numbers being in the same order as they appear on the number line, which is what akkarin wants.
– Dave L. Renfro
Apr 22 '16 at 17:52




@Matt Dyer: True, but the ordering you get may not have the numbers being in the same order as they appear on the number line, which is what akkarin wants.
– Dave L. Renfro
Apr 22 '16 at 17:52




2




2




Why the vote to close? It's possible this is a duplicate (I vaguely remember the same question being asked earlier), but it's certainly not off-topic.
– Noah Schweber
Apr 22 '16 at 18:52




Why the vote to close? It's possible this is a duplicate (I vaguely remember the same question being asked earlier), but it's certainly not off-topic.
– Noah Schweber
Apr 22 '16 at 18:52










1 Answer
1






active

oldest

votes

















up vote
6
down vote



accepted










There can't be. If $Ssubseteq Bbb R$ is well-ordered by the usual ordering, for every element $s_{alpha}in S$ that has an immediate successor $s_{alpha+1}in S$ (every element of $S$ except the greatest element if there is one), the set of rationals $Q_{alpha}$ between the element and its successor is nonempty: $(s_{alpha}, s_{alpha+1}) cap Bbb Q ne emptyset$, and the $Q_{alpha}$ are disjoint. If $S$ were uncountable, then $bigcup_{alpha < length(S)} Q_{alpha}$ would also be uncountable — impossible, as it's a subset of $Bbb Q$.






share|cite|improve this answer





















    Your Answer





    StackExchange.ifUsing("editor", function () {
    return StackExchange.using("mathjaxEditing", function () {
    StackExchange.MarkdownEditor.creationCallbacks.add(function (editor, postfix) {
    StackExchange.mathjaxEditing.prepareWmdForMathJax(editor, postfix, [["$", "$"], ["\\(","\\)"]]);
    });
    });
    }, "mathjax-editing");

    StackExchange.ready(function() {
    var channelOptions = {
    tags: "".split(" "),
    id: "69"
    };
    initTagRenderer("".split(" "), "".split(" "), channelOptions);

    StackExchange.using("externalEditor", function() {
    // Have to fire editor after snippets, if snippets enabled
    if (StackExchange.settings.snippets.snippetsEnabled) {
    StackExchange.using("snippets", function() {
    createEditor();
    });
    }
    else {
    createEditor();
    }
    });

    function createEditor() {
    StackExchange.prepareEditor({
    heartbeatType: 'answer',
    convertImagesToLinks: true,
    noModals: true,
    showLowRepImageUploadWarning: true,
    reputationToPostImages: 10,
    bindNavPrevention: true,
    postfix: "",
    imageUploader: {
    brandingHtml: "Powered by u003ca class="icon-imgur-white" href="https://imgur.com/"u003eu003c/au003e",
    contentPolicyHtml: "User contributions licensed under u003ca href="https://creativecommons.org/licenses/by-sa/3.0/"u003ecc by-sa 3.0 with attribution requiredu003c/au003e u003ca href="https://stackoverflow.com/legal/content-policy"u003e(content policy)u003c/au003e",
    allowUrls: true
    },
    noCode: true, onDemand: true,
    discardSelector: ".discard-answer"
    ,immediatelyShowMarkdownHelp:true
    });


    }
    });














    draft saved

    draft discarded


















    StackExchange.ready(
    function () {
    StackExchange.openid.initPostLogin('.new-post-login', 'https%3a%2f%2fmath.stackexchange.com%2fquestions%2f1754463%2fis-there-any-well-ordered-uncountable-set-of-real-numbers-under-the-original-ord%23new-answer', 'question_page');
    }
    );

    Post as a guest















    Required, but never shown

























    1 Answer
    1






    active

    oldest

    votes








    1 Answer
    1






    active

    oldest

    votes









    active

    oldest

    votes






    active

    oldest

    votes








    up vote
    6
    down vote



    accepted










    There can't be. If $Ssubseteq Bbb R$ is well-ordered by the usual ordering, for every element $s_{alpha}in S$ that has an immediate successor $s_{alpha+1}in S$ (every element of $S$ except the greatest element if there is one), the set of rationals $Q_{alpha}$ between the element and its successor is nonempty: $(s_{alpha}, s_{alpha+1}) cap Bbb Q ne emptyset$, and the $Q_{alpha}$ are disjoint. If $S$ were uncountable, then $bigcup_{alpha < length(S)} Q_{alpha}$ would also be uncountable — impossible, as it's a subset of $Bbb Q$.






    share|cite|improve this answer

























      up vote
      6
      down vote



      accepted










      There can't be. If $Ssubseteq Bbb R$ is well-ordered by the usual ordering, for every element $s_{alpha}in S$ that has an immediate successor $s_{alpha+1}in S$ (every element of $S$ except the greatest element if there is one), the set of rationals $Q_{alpha}$ between the element and its successor is nonempty: $(s_{alpha}, s_{alpha+1}) cap Bbb Q ne emptyset$, and the $Q_{alpha}$ are disjoint. If $S$ were uncountable, then $bigcup_{alpha < length(S)} Q_{alpha}$ would also be uncountable — impossible, as it's a subset of $Bbb Q$.






      share|cite|improve this answer























        up vote
        6
        down vote



        accepted







        up vote
        6
        down vote



        accepted






        There can't be. If $Ssubseteq Bbb R$ is well-ordered by the usual ordering, for every element $s_{alpha}in S$ that has an immediate successor $s_{alpha+1}in S$ (every element of $S$ except the greatest element if there is one), the set of rationals $Q_{alpha}$ between the element and its successor is nonempty: $(s_{alpha}, s_{alpha+1}) cap Bbb Q ne emptyset$, and the $Q_{alpha}$ are disjoint. If $S$ were uncountable, then $bigcup_{alpha < length(S)} Q_{alpha}$ would also be uncountable — impossible, as it's a subset of $Bbb Q$.






        share|cite|improve this answer












        There can't be. If $Ssubseteq Bbb R$ is well-ordered by the usual ordering, for every element $s_{alpha}in S$ that has an immediate successor $s_{alpha+1}in S$ (every element of $S$ except the greatest element if there is one), the set of rationals $Q_{alpha}$ between the element and its successor is nonempty: $(s_{alpha}, s_{alpha+1}) cap Bbb Q ne emptyset$, and the $Q_{alpha}$ are disjoint. If $S$ were uncountable, then $bigcup_{alpha < length(S)} Q_{alpha}$ would also be uncountable — impossible, as it's a subset of $Bbb Q$.







        share|cite|improve this answer












        share|cite|improve this answer



        share|cite|improve this answer










        answered Apr 22 '16 at 18:39









        BrianO

        14.3k1822




        14.3k1822






























            draft saved

            draft discarded




















































            Thanks for contributing an answer to Mathematics Stack Exchange!


            • Please be sure to answer the question. Provide details and share your research!

            But avoid



            • Asking for help, clarification, or responding to other answers.

            • Making statements based on opinion; back them up with references or personal experience.


            Use MathJax to format equations. MathJax reference.


            To learn more, see our tips on writing great answers.





            Some of your past answers have not been well-received, and you're in danger of being blocked from answering.


            Please pay close attention to the following guidance:


            • Please be sure to answer the question. Provide details and share your research!

            But avoid



            • Asking for help, clarification, or responding to other answers.

            • Making statements based on opinion; back them up with references or personal experience.


            To learn more, see our tips on writing great answers.




            draft saved


            draft discarded














            StackExchange.ready(
            function () {
            StackExchange.openid.initPostLogin('.new-post-login', 'https%3a%2f%2fmath.stackexchange.com%2fquestions%2f1754463%2fis-there-any-well-ordered-uncountable-set-of-real-numbers-under-the-original-ord%23new-answer', 'question_page');
            }
            );

            Post as a guest















            Required, but never shown





















































            Required, but never shown














            Required, but never shown












            Required, but never shown







            Required, but never shown

































            Required, but never shown














            Required, but never shown












            Required, but never shown







            Required, but never shown







            Popular posts from this blog

            To store a contact into the json file from server.js file using a class in NodeJS

            Redirect URL with Chrome Remote Debugging Android Devices

            Dieringhausen